please help me with that

Answers

Answer 1

Answer:

[tex]\frac{16}{81}[/tex]

Step-by-step explanation:

[tex](\frac{27}{8} )^{-\frac{4}{3} }[/tex]

[tex]=((\frac{3}{2} )^3)^{-4/3}[/tex]

[tex]=(\frac{3}{2} )^{-4}[/tex]

[tex]=(\frac{2}{3} )^{4}[/tex]

[tex]=\frac{16}{81}[/tex]

Answer 2

Answer:

16/81

Step-by-step explanation:

a negative exponent means 1/...

the number in the numerator means "to the power of".

the number in the denominator means take the root of that power.

so, we have to take the third root of the expression, or this then to the power of 4, and finally build 1/... if the whole result.

and the sequence is not making a difference.

the third root of of 27/8 = 3/2

this to the power of 4 = 81/16

this 1/... = 16/81


Related Questions

2. Determine the measure of the angles indicated by letters. Justify your answers with the
properties or theorems you used.

Answers

Answer:

a = 50°

b = 130°

c = 50°

d = 50°

e = 130°

f = 130°

g = 50°

Answered by GAUTHMATH

3,125 subtracted by what can give me 514

Answers

Answer:

2611

Step-by-step explanation:

3125-2611= 514

for more answers check my bio

PLS HELP ME ON THIS QUESTION I WILL MRK YOU AS BRAINLIEST IF YOU KNOW THE ANSWER!!
Which of the following measures is a measure of spread?
A. median
B. range
C. mode
D. mean

Answers

Answer:

range

Step-by-step explanation:

Answer:

B. range.

Step-by-step explanation:

others are:

» Standard variation.

» Interquatile range.

» Quatiles, deciles and percentiles.

» variance.

[tex]{ \underline{ \blue{ \sf{christ \: † \: alone}}}}[/tex]

Hi, Which option is correct??

Answers

Answer:

B

Step-by-step explanation:

option B is not similar.

the ratio of each side isn't same

3a + 2 = 20

5(b+1) = 10

3 (2y - 3) - 2y = y-3

2+ (2+4p) =6p

Please answer these questions with steps please!

Answers

1. 3a=20-2
3a=18
a=6

2. b+1=2
b=2-1
b=1

3. 6y-9-2y=y-3
4y-9=y-3
4y-y=-3+9
3y=6
y=2

4. 2+2+4p=6p
4+4p=6p
4p-6p=-4
-2p=-4
p=2

F is on the bisector of angle BCD. Find the length of FD (with lines over FD)

Answers

Answer:

8n-2 = 6n+9

2n-2 = 9

2n = 11

n = 5.5

So C is correct

Let me know if this helps!

how many inches is 775 centimeters

Answers

Answer:

305.11

Step-by-step explanation:

Just use a calculator. A centimeter is 2.5 inches. Divide 775 by that.

What is the area of a rectangle with vertices at (7,3) (12,3) (12,11) (7,11)

Answers

Answer:

Area = 5 × 8

= 40 square units

Answer:

40^2

Step by Step Solution:

I counted the difference between the length and the width, which was 5 and 8, then using the formula for area, lw=a^2, I did 5(8)=40^2.  Some people leave out the squared part of the area, but 40^2 would be the most correct option if they do not square any of the answers, just put 40 that'll probably be accepted too.

ax^2-y^2-x-y factorize​

Answers

Answer:

x(ax-1)-y(y+1)

Step-by-step explanation:

you have to group the like terms

ax^2-x-y^2-y

x(ax-1)-y(y+1)

I hope this helps

If LM = 9x + 27 and RS = 135, find x.

Answers

Answer:

x=12

Step-by-step explanation:

LM = RS

9x+27 = 135

Subtract 27 from each side

9x+27-27 =135-27

9x=108

Divide each side by 9

9x/9 = 108/9

x = 12

the boxes are equivalent so the one with a single dash is equal to the other with a single dash.

the one with 2 dashes is equal to the other with 2 dashes so on and so forth

SR=LM

LM=9x+27

RS=135

9x+27=135

so I solve it in my own weird way but you can solve it differently. 135-27=108

108/9=12

so your answer is 12

Two observers are 300 ft apart on opposite sides of a flagpole. The angles of
elevation from the observers to the top of the pole are 20°
and 15°. Find the
height of the flagpole.

Answers

I know similar questions and have answers. do you want

Which value of x makes this equation true?-9x+15=3(2-x)

Answers

Step-by-step explanation:

-9x+15=3(2-x)

expand the bracket by the right hand side

6-6x

2. collect like terms

-9x+15= 6-6x

15-6 = 6x+9x

11= 15x

3. divide both sides by the coefficient of X which is 15

x= 11/15

If a line has a midpoint at (2,5), and the endpoints are (0,0) and (4,y), what is the value of y? Please explain each step for a better understanding:)

Answers

Answer:

y = 10

Step-by-step explanation:

To find the y coordinate of the midpoint, take the y coordinates of the endpoints and average

(0+y)/2 = 5

Multiply each  die by 2

0+y = 10

y = 10

result of 5 and 75 with dividid by 3

Answers

Answer:

your answer is 30

Step-by-step explanation:

I hope this help

Determine three consecutive odd integers whose sum is 2097.

Answers

Answer:

first odd integer=x

second odd integer=x+2

third odd integer=x+4

x+x+2+x+4=2097

x+x+x+2+4=2097

3x+6=2097

3x=2097-6

3x=2091

3x/3=2091/3

x=697

therefore, x=697

x+2=697+2=699

x+4=697+4=701

PLEASE HELP I WILL GIVE BRAINLIEST

Answers

Step-by-step explanation:

A natural number is a positive whole number.

A whole number is a positive number with no fractions or decimals.

A interger is a whole number negative or positive.

A rational number is a number that terminates or continue with repeating digits.

A irrational number is a number that doesn't terminate or continue with repeating digits.

1. Rational Number

2. Natural,Whole,Interger,Rational

3. Whole,Rational,Interger

4. Rational

5.Irrational

6.Rational

7.Natural,Whole,Interger,Rational

8.Interger,Rational

9.Irrational


Rationalise the denominator

Answers

Answer:

sqrt(3) /3

Step-by-step explanation:

1 / sqrt(3)

Multiply the top and bottom by sqrt(3)

1/ sqrt(3) * sqrt(3)/ sqrt(3)

sqrt(3) /  sqrt(3)*sqrt(3)

sqrt(3) /3

Answer:

[tex] = { \sf{ \frac{1}{ \sqrt{3} } }} \\ \\ { \sf{ = \frac{1}{ \sqrt{3} } . \frac{ \sqrt{3} }{ \sqrt{3} } }} \\ \\ = { \sf{ \frac{ \sqrt{3} }{ {( \sqrt{3}) }^{2} } = \frac{ \sqrt{3} }{3} }} [/tex]

Write the equation of the sinusoidal function shown?

A) y = cos x + 2

B) y = cos(3x) + 2

C) y = sin x + 2

D) y = sin(3x) + 2

Answers

Answer:

günah(3x) + 2

Step-by-step explanation:

Gösterilen sinüzoidal fonksiyonun denklemini yazınız? A) y = cos x + 2 B) y = cos(3x) + 2 C) y = günah x + 2 D) y =

Answer:

y = sin(3x) + 2

Where did term “infinity” come from

Answers

the English mathematician John Wallis in 1655 invented the word infinity Infinity is from the Latin, infinitas. In general, the word signifies the state from an entity's not ending/limit.

convert 10.09% to a decimal

Answers

Answer:

0.1009

Step-by-step explanation:

To convert percentage into decimal, you need to divide the percentage by 100

10.09/100

= 0.1009

To convert 10.09% to a decimal, we need to decide it by 100 like so:

10.09 ÷ 100 = 0.1009

Therefore, the answer is 0.1009

Find the value of the sum 219+226+233+⋯+2018.

Assume that the terms of the sum form an arithmetic series.

Give the exact value as your answer, do not round.

Answers

Answer:

228573

Step-by-step explanation:

a = 219 (first term)

an = 2018 (last term)

Sn->Sum of n terms

Sn=n/2(a + an)         [Where n is no. of terms] -> eq 1

To find number of terms,

an = a + (n-1)d     [d->Common Difference] -> eq 2

d= 226-219 = 7

=> d=7

Substituting in eq 2,

2018 = 219 + (n-1)(7)

1799 = (n-1)(7)

1799 = 7n-7

1799 = 7(n-1)

1799/7 = n-1

257 = n-1

n=258

Substituting values in eq 1,

Sn = 258/2(219+2018)

    = 129(2237)

    = 228573

Determine the sum of the first 33 terms of the following series:

−52+(−46)+(−40)+...

Answers

Answer:

1320

Step-by-step explanation:

Use the formula for sum of series, s(a) = n/2(2a + (n-1)d)

The terms increase by 6, so d is 6

a is the first term, -56

n is the terms you want to find, 33

Plug in the numbers, 33/2 (2(-56)+(32)6)

Simplify into 33(80)/2 and you get 1320

help help help help

Answers

Answer:

abc is a triangle so ,

a is ( 9,6 )

b is ( 9,3 )

and c is ( 3,3 )

Please help I’ll mark as brainlist

Answers

Answer:

Ekta and Preyal

Step-by-step explanation:

Answer: Ekta and Preyal

Originally the cubes have a perimeter of 15, both Ekta and Preyal have a perimeter of 17 which is exactly a 2 unit increase

−30=5(x+1)

what is x?

Answers

[tex]\\ \rm\Rrightarrow -30=5(x+1)[/tex]

[tex]\\ \rm\Rrightarrow -30=5x+5[/tex]

[tex]\\ \rm\Rrightarrow 5x=-30-5[/tex]

[tex]\\ \rm\Rrightarrow 5x=-35[/tex]

[tex]\\ \rm\Rrightarrow x=\dfrac{-35}{-5}[/tex]

[tex]\\ \rm\Rrightarrow x=7[/tex]

Answer:

x = -7

Step-by-step explanation:

-30 = 5 (x -1 )

5 ( x + 1 ) =-30

5 (x + 1 ) = - 30

     5            5

x + 1 = -6

x + 1 -1 = -6 -1

x = - 7

100 POINTS AND BRAINLIEST FOR THIS WHOLE SEGMENT

a) Find zw, Write your answer in both polar form with ∈ [0, 2pi] and in complex form.

b) Find z^10. Write your answer in both polar form with ∈ [0, 2pi] and in complex form.

c) Find z/w. Write your answer in both polar form with ∈ [0, 2pi] and in complex form.

d) Find the three cube roots of z in complex form. Give answers correct to 4 decimal

places.

Answers

Answer:

See Below (Boxed Solutions).

Step-by-step explanation:

We are given the two complex numbers:

[tex]\displaystyle z = \sqrt{3} - i\text{ and } w = 6\left(\cos \frac{5\pi}{12} + i\sin \frac{5\pi}{12}\right)[/tex]

First, convert z to polar form. Recall that polar form of a complex number is:

[tex]z=r\left(\cos \theta + i\sin\theta\right)[/tex]

We will first find its modulus r, which is given by:

[tex]\displaystyle r = |z| = \sqrt{a^2+b^2}[/tex]

In this case, a = √3 and b = -1. Thus, the modulus is:

[tex]r = \sqrt{(\sqrt{3})^2 + (-1)^2} = 2[/tex]

Next, find the argument θ in [0, 2π). Recall that:

[tex]\displaystyle \tan \theta = \frac{b}{a}[/tex]

Therefore:

[tex]\displaystyle \theta = \arctan\frac{(-1)}{\sqrt{3}}[/tex]

Evaluate:

[tex]\displaystyle \theta = -\frac{\pi}{6}[/tex]

Since z must be in QIV, using reference angles, the argument will be:

[tex]\displaystyle \theta = \frac{11\pi}{6}[/tex]

Therefore, z in polar form is:

[tex]\displaystyle z=2\left(\cos \frac{11\pi}{6} + i \sin \frac{11\pi}{6}\right)[/tex]

Part A)

Recall that when multiplying two complex numbers z and w:

[tex]zw=r_1\cdot r_2 \left(\cos (\theta _1 + \theta _2) + i\sin(\theta_1 + \theta_2)\right)[/tex]

Therefore:

[tex]\displaystyle zw = (2)(6)\left(\cos\left(\frac{11\pi}{6} + \frac{5\pi}{12}\right) + i\sin\left(\frac{11\pi}{6} + \frac{5\pi}{12}\right)\right)[/tex]

Simplify. Hence, our polar form is:

[tex]\displaystyle\boxed{zw = 12\left(\cos\frac{9\pi}{4} + i\sin \frac{9\pi}{4}\right)}[/tex]

To find the complex form, evaluate:

[tex]\displaystyle zw = 12\cos \frac{9\pi}{4} + i\left(12\sin \frac{9\pi}{4}\right) =\boxed{ 6\sqrt{2} + 6i\sqrt{2}}[/tex]

Part B)

Recall that when raising a complex number to an exponent n:

[tex]\displaystyle z^n = r^n\left(\cos (n\cdot \theta) + i\sin (n\cdot \theta)\right)[/tex]

Therefore:

[tex]\displaystyle z^{10} = r^{10} \left(\cos (10\theta) + i\sin (10\theta)\right)[/tex]

Substitute:

[tex]\displaystyle z^{10} = (2)^{10} \left(\cos \left(10\left(\frac{11\pi}{6}\right)\right) + i\sin \left(10\left(\frac{11\pi}{6}\right)\right)\right)[/tex]

Simplify:

[tex]\displaystyle z^{10} = 1024\left(\cos\frac{55\pi}{3}+i\sin \frac{55\pi}{3}\right)[/tex]

Simplify using coterminal angles. Thus, the polar form is:

[tex]\displaystyle \boxed{z^{10} = 1024\left(\cos \frac{\pi}{3} + i\sin \frac{\pi}{3}\right)}[/tex]

And the complex form is:

[tex]\displaystyle z^{10} = 1024\cos \frac{\pi}{3} + i\left(1024\sin \frac{\pi}{3}\right) = \boxed{512+512i\sqrt{3}}[/tex]

Part C)

Recall that:

[tex]\displaystyle \frac{z}{w} = \frac{r_1}{r_2} \left(\cos (\theta_1-\theta_2)+i\sin(\theta_1-\theta_2)\right)[/tex]

Therefore:

[tex]\displaystyle \frac{z}{w} = \frac{(2)}{(6)}\left(\cos \left(\frac{11\pi}{6} - \frac{5\pi}{12}\right) + i \sin \left(\frac{11\pi}{6} - \frac{5\pi}{12}\right)\right)[/tex]

Simplify. Hence, our polar form is:

[tex]\displaystyle\boxed{ \frac{z}{w} = \frac{1}{3} \left(\cos \frac{17\pi}{12} + i \sin \frac{17\pi}{12}\right)}[/tex]

And the complex form is:

[tex]\displaystyle \begin{aligned} \frac{z}{w} &= \frac{1}{3} \cos\frac{5\pi}{12} + i \left(\frac{1}{3} \sin \frac{5\pi}{12}\right)\right)\\ \\ &=\frac{1}{3}\left(\frac{\sqrt{2}-\sqrt{6}}{4}\right) + i\left(\frac{1}{3}\left(- \frac{\sqrt{6} + \sqrt{2}}{4}\right)\right) \\ \\ &= \boxed{\frac{\sqrt{2} - \sqrt{6}}{12} -\frac{\sqrt{6}+\sqrt{2}}{12}i}\end{aligned}[/tex]

Part D)

Let a be a cube root of z. Then by definition:

[tex]\displaystyle a^3 = z = 2\left(\cos \frac{11\pi}{6} + i\sin \frac{11\pi}{6}\right)[/tex]

From the property in Part B, we know that:

[tex]\displaystyle a^3 = r^3\left(\cos (3\theta) + i\sin(3\theta)\right)[/tex]

Therefore:

[tex]\displaystyle r^3\left(\cos (3\theta) + i\sin (3\theta)\right) = 2\left(\cos \frac{11\pi}{6} + i\sin \frac{11\pi}{6}\right)[/tex]

If two complex numbers are equal, their modulus and arguments must be equivalent. Thus:

[tex]\displaystyle r^3 = 2\text{ and } 3\theta = \frac{11\pi}{6}[/tex]

The first equation can be easily solved:

[tex]r=\sqrt[3]{2}[/tex]

For the second equation, 3θ must equal 11π/6 and any other rotation. In other words:

[tex]\displaystyle 3\theta = \frac{11\pi}{6} + 2\pi n\text{ where } n\in \mathbb{Z}[/tex]

Solve for the argument:

[tex]\displaystyle \theta = \frac{11\pi}{18} + \frac{2n\pi}{3} \text{ where } n \in \mathbb{Z}[/tex]

There are three distinct solutions within [0, 2π):

[tex]\displaystyle \theta = \frac{11\pi}{18} , \frac{23\pi}{18}\text{ and } \frac{35\pi}{18}[/tex]

Hence, the three roots are:

[tex]\displaystyle a_1 = \sqrt[3]{2} \left(\cos\frac{11\pi}{18}+ \sin \frac{11\pi}{18}\right) \\ \\ \\ a_2 = \sqrt[3]{2} \left(\cos \frac{23\pi}{18} + i\sin\frac{23\pi}{18}\right) \\ \\ \\ a_3 = \sqrt[3]{2} \left(\cos \frac{35\pi}{18} + i\sin \frac{35\pi}{18}\right)[/tex]

Or, approximately:

[tex]\displaystyle\boxed{ a _ 1\approx -0.4309 + 1.1839i,} \\ \\ \boxed{a_2 \approx -0.8099-0.9652i,} \\ \\ \boxed{a_3\approx 1.2408-0.2188i}[/tex]

help me pls??????? :)

Answers

Answer:4 in each bad 2 left over

Step-by-step explanation:

Answer:

4 in each bag and 2 left over

Step-by-step explanation:

divide 14 by 3

3 goes into 14, 4 times

14 - 12 = 2

4 in each bag and then 2 left over

Select the correct answer from each drop-down menu.
A company makes cylindrical vases. The capacity, in cubic centimeters, of a cylindrical vase the company produces is given by the
function C() = 6.2873 + 28.26x2, where x is the radius, in centimeters. The area of the circular base of a vase, in square
centimeters, is given by the function A () = 3.14.2
To find the height of the vase, divide
represents the height of the vase.
the expressions modeling functions C(x) and A(z). The expression

Answers

Answer:

divide, 2x+9

Step-by-step explanation:

got it right

plzzzz heeeeeeellllllllppppppppp again...

Answers

ANS=40

hope this help you

bye have a great day :)

180-55=25
the interior angle of triangle is 180
so 180-115-25= 40
the answer is 40
give me brainliest if you can thank u!:)

Pls help it’s due in the morning ;(

Answers

9:-

(3,3)(-4,1)

[tex]\\ \sf\longmapsto m=\dfrac{y_2-y_1}{x_2-x_1}[/tex]

[tex]\\ \sf\longmapsto m=\dfrac{1-3}{-4-3}[/tex]

[tex]\\ \sf\longmapsto m=\dfrac{-2}{-7}[/tex]

[tex]\\ \sf\longmapsto m=\dfrac{2}{7}[/tex]

10:-

Points are (-7,6),(11,-4)

[tex]\boxed{\sf slope(m)=\dfrac{y_2-y_1}{x_2-x_1}}[/tex]

[tex]\\ \sf\longmapsto m=\dfrac{-4-6}{11+7}[/tex]

[tex]\\ \sf\longmapsto m=\dfrac{-10}{18}[/tex]

[tex]\\ \sf\longmapsto m=-\dfrac{5}{9}[/tex]

Answer:

Step-by-step explanation:

Slope = [tex]\frac{y_{2}-y_{1}}{x_{2}-x_{1}}[/tex]

9) Mark any two point on the line

(x₁ , y₁) = (3 , 3)   ;   (x₂, y₂) = (-4 ,1)

[tex]Slope =\frac{1-3}{-4-3}\\\\=\frac{-2}{-7}\\\\=\frac{2}{7}[/tex]

10) (x₁ , y₁) = ( -7 , 6)   ;   (x₂, y₂) = (11 ,-4)

[tex]Slope =\frac{-4-6}{11-[-7]}\\\\ =\frac{-4-6}{11+7}\\\\=\frac{-10}{18}\\\\=\frac{-5}{9}[/tex]

Other Questions
Which of the binomials below is a factor of this expression?16x2 + 40xy + 25y2 Figure AFigure BHow many squarw many square are intignethis Please help me with this on the image 3 log2 (x+1) - 2 = 13 The number of organic compounds that have been reported in 2019 is more than 10 million. This is due to carbon's ability to bond to other carbon molecules, calledelectron affinity.ionic bonding.catenation.electronegativity. 2. Determine the measure of the angles indicated by letters. Justify your answers with theproperties or theorems you used. ASAP What occurs after the homologous pairs are separated?A. The sister chromatids separate.B. Spindle fibers attach to the chromosomes.C. Haploid cells are formed.D. The nuclear envelope reforms. Which of the following groups is the largest ? population community ecosystem biome Use the point-slope formula to determine the equation of the line that has a slope of 12 and passes through point (0, 0). 3,125 subtracted by what can give me 514 what should we do to the violators of social rules ? Suggest any two measures. According to a study published in Conservation Letters,a research team led by biologist Brian MacKenzie ran computer models of the population dynamics of the bluefin tuna that suggest that,even if fishing were banned immediately,the population of bluefin in the Atlantic and Mediterranean oceans will most likely collapse.This is a serious issue for sushi lovers everywhere,and some sociologists call it: A) technological diffusion. B) ecoterrorism. C) a tragedy of the commons. D) a public goods dilemma. Two sides of a triangle have lengths 13 m and 19 m. The angle between them is increasing at a rate of 2/min. How fast is the length of the third side increasing when the angle between the sides of fixed length is 60? (Round your answer to three decimal places.) The author mentions the fire train to In measuring GDP as the sum of all incomes earned in production, we should include: purchases of used goods. purchases of used goods. American purchases of foreign output. American purchases of foreign output. wages and salaries. wages and salaries. For the reaction 2HNO3 + Mg(OH)2 + Mg(NO3)2 + 2H20, how manygrams of magnesium nitrate are produced from 3.54 mol of nitricacid, HNO3?O a. 220O b. 263O c. 1050O d. 175 abolt from the blue, PLS HELP ME ON THIS QUESTION I WILL MRK YOU AS BRAINLIEST IF YOU KNOW THE ANSWER!!Which of the following measures is a measure of spread?A. medianB. rangeC. modeD. mean Hi, Which option is correct?? 3a + 2 = 20 5(b+1) = 103 (2y - 3) - 2y = y-32+ (2+4p) =6pPlease answer these questions with steps please!